Vector Algebra

Short Answer Type Questions

1. Find the unit vector in the direction of sum of vectors $\vec{a}=2 \hat{i}-\hat{j}+\hat{k}$ and $\vec{b}=2 \hat{j}+\hat{k}$.

Show Answer

Solution

Given that

$ \begin{aligned} \vec{a} & =2 \hat{i}-\hat{j}+\hat{k} \text{ and } \vec{b}=2 \hat{j}+\hat{k} \\ \vec{a}+\vec{b} & =(2 \hat{i}-\hat{j}+\hat{k})+(2 \hat{j}+\hat{k})=2 \hat{i}+\hat{j}+2 \hat{k} \end{aligned} $

$\therefore \quad$ Unit vector in the direction of $\vec{a}+\vec{b}=\frac{\vec{a}+\vec{b}}{|\vec{a}+\vec{b}|}$

$ \begin{aligned} & =\frac{2 \hat{i}+\hat{j}+2 \hat{k}}{\sqrt{(2)^{2}+(1)^{2}+(2)^{2}}}=\frac{2 \hat{i}+\hat{j}+2 \hat{k}}{\sqrt{4+1+4}} \\ & =\frac{2 \hat{i}+\hat{j}+2 \hat{k}}{\sqrt{9}}=\frac{2 \hat{i}+\hat{j}+2 \hat{k}}{3}=\frac{2}{3} \hat{i}+\frac{1}{3} \hat{j}+\frac{2}{3} \hat{k} \end{aligned} $

Hence, the required unit vector is $\frac{2}{3} \hat{i}+\frac{1}{3} \hat{j}+\frac{2}{3} \hat{k}$.

2. If $\vec{a}=\hat{i}+\hat{j}+2 \hat{k}$ and $\vec{b}=2 \hat{i}+\hat{j}-2 \hat{k}$, find the unit vector in the direction of

(i) $6 \vec{b}$

(ii) $2 \vec{a}-\vec{b}$

Show Answer

Solution

Given that $\vec{a}=\hat{i}+\hat{j}+2 \hat{k}$ and $\vec{b}=2 \hat{i}+\hat{j}-2 \hat{k}$

(i) $6 \vec{b}=6(2 \hat{i}+\hat{j}-2 \hat{k})=12 \hat{i}+6 \hat{j}-12 \hat{k}$

$\therefore \quad$ Unit vector in the direction of $6 \vec{b}=\frac{6 \vec{b}}{|6 \vec{b}|}$

$ \begin{aligned} & =\frac{12 \hat{i}+6 \hat{j}-12 \hat{k}}{\sqrt{(12)^{2}+(6)^{2}+(-12)^{2}}}=\frac{12 \hat{i}+6 \hat{j}-12 \hat{k}}{\sqrt{144+36+144}} \\ & =\frac{12 \hat{i}+6 \hat{j}-12 \hat{k}}{\sqrt{324}}=\frac{12 \hat{i}+6 \hat{j}-12 \hat{k}}{18} \\ & =\frac{6}{18}(2 \hat{i}+\hat{j}-2 \hat{k})=\frac{1}{3}(2 \hat{i}+\hat{j}-2 \hat{k}) \end{aligned} $

Hence, the required unit vector is $\frac{1}{3}(2 \hat{i}+\hat{j}-2 \hat{k})$.

(ii) $2 \vec{a}-\vec{b}=2(\hat{i}+\hat{j}+2 \hat{k})-(2 \hat{i}+\hat{j}-2 \hat{k})$

$ =2 \hat{i}+2 \hat{j}+4 \hat{k}-2 \hat{i}-\hat{j}+2 \hat{k}=\hat{j}+6 \hat{k} $

$\therefore \quad$ Unit vector in the direction of $2 \vec{a}-\vec{b}$

$ \begin{aligned} & =\frac{2 \vec{a}-\vec{b}}{|2 \vec{a}-\vec{b}|}=\frac{\hat{j}+6 \hat{k}}{\sqrt{(1)^{2}+(6)^{2}}}=\frac{\hat{j}+6 \hat{k}}{\sqrt{1+36}} \\ & =\frac{\hat{j}+6 \hat{k}}{\sqrt{37}}=\frac{1}{\sqrt{37}}[\hat{j}+6 \hat{k}] \end{aligned} $

Hence, the required unit vector is $\frac{1}{\sqrt{37}}[\hat{j}+6 \hat{k}]$.

3. Find a unit vector in the direction of $\overrightarrow{{}PQ}$, where $P$ and $Q$ have coordinates $(5,0,8)$ and $(3,3,2)$ respectively.

Show Answer

Solution

Given coordinates are $P(5,0,8)$ and $Q(3,3,2)$

$\therefore \quad \overrightarrow{{}PQ}=(3-5) \hat{i}+(3-0) \hat{j}+(2-8) \hat{k}=-2 \hat{i}+3 \hat{j}-6 \hat{k}$

$\therefore \quad$ Unit vector in the direction of $\overrightarrow{{}PQ}=\frac{\overrightarrow{{}PQ}}{|\overrightarrow{{}PQ}|}$

$ \begin{aligned} & =\frac{-2 \hat{i}+3 \hat{j}-6 \hat{k}}{\sqrt{(-2)^{2}+(3)^{2}+(-6)^{2}}}=\frac{-2 \hat{i}+3 \hat{j}-6 \hat{k}}{\sqrt{4+9+36}}=\frac{-2 \hat{i}+3 \hat{j}-6 \hat{k}}{\sqrt{49}} \\ & =\frac{-2 \hat{i}+3 \hat{j}-6 \hat{k}}{7}=\frac{1}{7}(-2 \hat{i}+3 \hat{j}-6 \hat{k}) \end{aligned} $

Hence, the required unit vector is $\frac{1}{7}(-2 \hat{i}+3 \hat{j}-6 \hat{k})$.

4. If $\vec{a}$ and $\vec{b}$ are the position vectors of $A$ and $B$ respectively, find the position vector of a point $C$ in $B A$ produced such that $BC=1.5 BA$.

Show Answer

Solution

Given that

$ \begin{matrix} BC =1.5 BA \\ \Rightarrow \quad \frac{BC}{BA} =1.5=\frac{3}{2} \\ \Rightarrow \quad \frac{\vec{c}-\vec{b}}{\vec{a}-\vec{b}} =\frac{3}{2} \\ \Rightarrow 2 \vec{c}-2 \vec{b} =3 \vec{a}-3 \vec{b} \Rightarrow 2 \vec{c}=3 \vec{a}-3 \vec{b}+2 \vec{b} \Rightarrow 2 \vec{c}=3 \vec{a}-\vec{b} \\ \therefore \quad \vec{c} =\frac{3 \vec{a}-\vec{b}}{2} \end{matrix} $

Hence, the required vector is $\vec{c}=\frac{3 \vec{a}-\vec{b}}{2}$.

5. Using vectors, find the value of $k$, such that the points $(k,-10,3),(1,-1,3)$ and $(3,5,3)$ are collinear.

Show Answer

Solution

Let the given points are $A(k,-10,3), B(1,-1,3)$ and $C(3,5,3)$

$ \begin{aligned} \overrightarrow{{}AB} & =(1-k) \hat{i}+(-1+10) \hat{j}+(3-3) \hat{k} \\ \overrightarrow{{}AB} & =(1-k) \hat{i}+9 \hat{j}+0 \hat{k} \\ \therefore \mid \overrightarrow{{}AB} & =\sqrt{(1-k)^{2}+(9)^{2}}=\sqrt{(1-k)^{2}+81} \\ \overrightarrow{{}BC} & =(3-1) \hat{i}+(5+1) \hat{j}+(3-3) \hat{k}=2 \hat{i}+6 \hat{j}+0 \hat{k} \\ \therefore \mid \overrightarrow{{}BC} & =\sqrt{(2)^{2}+(6)^{2}}=\sqrt{4+36}=\sqrt{40}=2 \sqrt{10} \\ \overrightarrow{{}AC} & =(3-k) \hat{i}+(5+10) \hat{j}+(3-3) \hat{k}=(3-k) \hat{i}+15 \hat{j}+0 \hat{k} \\ \therefore \quad|\overrightarrow{{}AC}| & =\sqrt{(3-k)^{2}+(15)^{2}}=\sqrt{(3-k)^{2}+225} \end{aligned} $

If $A, B$ and $C$ are collinear, then

$ \begin{aligned} |\overrightarrow{{}AB}|+|\overrightarrow{{}BC}| & =|\overrightarrow{{}AC}| \\ \sqrt{(1-k)^{2}+81}+\sqrt{40} & =\sqrt{(3-k)^{2}+225} \end{aligned} $

Squaring both sides, we have

${[\sqrt{(1-k)^{2}+81}+\sqrt{40}]^{2}=[\sqrt{(3-k)^{2}+225}]^{2}}$

$\Rightarrow (1-k)^{2}+81+40+2 \sqrt{40} \sqrt{(1-k)^{2}+81}=(3-k)^{2}+225$

$\Rightarrow 1+k^{2}-2 k+121+2 \sqrt{40} \sqrt{1+k^{2}-2 k+81} =9+k^{2}-6 k+225$

$\Rightarrow 122-2 k+2 \sqrt{40} \sqrt{k^{2}-2 k+82}=234-6 k$

Dividing by 2 , we get

$ \begin{matrix} \Rightarrow & 61-k+\sqrt{40} \sqrt{k^{2}-2 k+82}=117-3 k \\ \Rightarrow & \sqrt{40} \sqrt{k^{2}-2 k+82}=117-61-3 k+k \\ \Rightarrow & \sqrt{40} \sqrt{k^{2}-2 k+82}=56-2 k \Rightarrow 2 \sqrt{10} \sqrt{k^{2}-2 k+82}=56-2 k \\ \Rightarrow & \sqrt{10} \sqrt{k^{2}-2 k+82}=28-k \quad \text{ (Dividing by } 2 \text{ ) } \end{matrix} $

Squaring both sides, we get

$ \begin{aligned} & \Rightarrow \quad 10(k^{2}-2 k+82)=784+k^{2}-56 k \\ & \Rightarrow \quad 10 k^{2}-20 k+820=784+k^{2}-56 k \\ & \Rightarrow \quad 10 k^{2}-k^{2}-20 k+56 k+820-784=0 \\ & \Rightarrow 9 k^{2}+36 k+36=0 \Rightarrow k^{2}+4 k+4=0 \Rightarrow(k+2)^{2}=0 \end{aligned} $

$ \Rightarrow \quad k+2=0 \quad \Rightarrow k=-2 $

Hence, the required value is $k=-2$

6. A vector $\vec{r}$ is inclined at equal angles to the three axes. If the magnitude of $\vec{r}$ is $2 \sqrt{3}$ units, find $\vec{r}$.

Show Answer

Solution

Since, the vector $\vec{r}$ makes equal angles with the axes, their direction cosines should be same

$\therefore \quad l=m=n$

We know that $\quad l^{2}+m^{2}+n^{2}=1 \quad \Rightarrow \quad l^{2}+l^{2}+l^{2}=1$

$ \begin{matrix} \Rightarrow \quad 3 l^{2}=1 \Rightarrow l^2=\frac{1}{3} \Rightarrow l=\pm \frac{1}{\sqrt {3}} \\ \quad \hat{r}= \pm \frac{1}{\sqrt{3}} \hat{ i }\pm \frac{1}{\sqrt{3}} \hat{j} \pm \frac{1}{\sqrt{3}} \hat{k} \Rightarrow \hat{r}= \pm \frac{1}{\sqrt{3}}(\hat{i}+\hat{j}+\hat{k}) \\ \end{matrix} $

We know that $\vec{r}=(\hat{r})|\vec{r}|$

$ = \pm \frac{1}{\sqrt{3}}(\hat{i}+\hat{j}+\hat{k}) 2 \sqrt{3}= \pm 2(\hat{i}+\hat{j}+\hat{k}) $

Hence, the required value of $\vec{r}$ is $\pm 2(\hat{i}+\hat{j}+\hat{k})$.

7. A vector $\vec{r}$ has magnitude 14 and direction ratios 2, 3 and -6 . Find the direction cosines and components of $\vec{r}$, given that $\vec{r}$ makes an acute angle with $x$-axis.

Show Answer

Solution

Let $\vec{a}, \vec{b}$ and $\vec{c}$ be three vectors such that $\vec{a}=2 k, \vec{b}=3 k$ and $\vec{c}=-6 k$

If $l, m$ and $n$ are the direction cosines of vector $\vec{r}$, then

$ \begin{aligned} l & =\frac{\vec{a}}{|\vec{r}|}=\frac{2 k}{14}=\frac{k}{7} \\ m & =\frac{\vec{b}}{|\vec{r}|}=\frac{3 k}{14} \text{ and } n=\frac{\vec{c}}{|\vec{r}|}=\frac{-6 k}{14}=\frac{-3 k}{7} \end{aligned} $

We know that $l^{2}+m^{2}+n^{2}=1$

$ \begin{matrix} \therefore & \frac{k^{2}}{49}+\frac{9 k^{2}}{196}+\frac{9 k^{2}}{49}=1 \\ \Rightarrow & \frac{4 k^{2}+9 k^{2}+36 k^{2}}{196}=1 \Rightarrow 49 k^{2}=196 \Rightarrow k^{2}=4 \\ \therefore & k= \pm 2 \text{ and } l=\frac{k}{7}=\frac{2}{7} \\ & m=\frac{3 k}{14}=\frac{3 \times 2}{14}=\frac{3}{7} \text{ and } n=\frac{-3 k}{7} \frac{-3 \times 2}{7}=\frac{-6}{7} \\ & \hat{r}= \pm(\frac{2}{7} \hat{i}+\frac{3}{7} \hat{j}-\frac{6}{7} \hat{k}) \end{matrix} $

$ \begin{aligned} \hat{r} & =\hat{r}|\vec{r}| \\ \Rightarrow \quad \vec{r} & = \pm(\frac{2}{7} \hat{i}+\frac{3}{7} \hat{j}-\frac{6}{7} \hat{k}) \cdot 14= \pm(4 \hat{i}+6 \hat{j}-12 \hat{k}) \end{aligned} $

Hence, the required direction cosines are $\frac{2}{7}, \frac{3}{7}, \frac{-6}{7}$ and the components of $\vec{r}$ are $4 \hat{i}, 6 \hat{j}$ and $-12 \hat{k}$.

8. Find a vector of magnitude 6 , which is perpendicular to both the vectors $2 \hat{i}-\hat{j}+2 \hat{k}$ and $4 \hat{i}-\hat{j}+3 \hat{k}$.

Show Answer

Solution

Let $\vec{a}=2 \hat{i}-\hat{j}+2 \hat{k}$ and $\vec{b}=4 \hat{i}-\hat{j}+3 \hat{k}$

We know that unit vector perpendicular to $\vec{a}$ and $\vec{b}=\frac{(\vec{a} \times \vec{b})}{|\vec{a} \times \vec{b}|}$

$ \begin{aligned} \vec{a} \times \vec{b} & = \begin{vmatrix} \hat{i} & \hat{j} & \hat{k} \\ 2 & -1 & 2 \\ 4 & -1 & 3 \end{vmatrix} \\ & =\hat{i}(-3+2)-\hat{j}(6-8)+\hat{k}(-2+4)=-\hat{i}+2 \hat{j}+2 \hat{k} \end{aligned} $

$\therefore \quad|\vec{a} \times \vec{b}|=\sqrt{(-1)^{2}+(2)^{2}+(2)^{2}}=\sqrt{1+4+4}=\sqrt{9}=3$

so, $\quad \frac{(\vec{a} \times \vec{b})}{|\vec{a} \times \vec{b}|}=\frac{-\hat{i}+2 \hat{j}+2 \hat{k}}{3}=\frac{1}{3}(-\hat{i}+2 \hat{j}+2 \hat{k})$

Now the vector of magnitude $6=\frac{1}{3}(-\hat{i}+2 \hat{j}+2 \hat{k}) \cdot 6$

$ =2(-\hat{i}+2 \hat{j}+2 \hat{k})=-2 \hat{i}+4 \hat{j}+4 \hat{k} $

Hence, the required vector is $-2 \hat{i}+4 \hat{j}+4 \hat{k}$.

9. Find the angle between the vectors $2 \hat{i}-\hat{j}+\hat{k}$ and $3 \hat{i}+4 \hat{j}-\hat{k}$.

Show Answer

Solution

Let $\vec{a}=2 \hat{i}-\hat{j}+\hat{k}$ and $\vec{b}=3 \hat{i}+4 \hat{j}-\hat{k}$

and let $\theta$ be the angle between $\vec{a}$ and $\vec{b}$.

$ \begin{aligned} \therefore \quad \cos \theta & =\frac{\vec{a} \cdot \vec{b}}{|\vec{a}||\vec{b}|}=\frac{(2 \hat{i}-\hat{j}+\hat{k}) \cdot(3 \hat{i}+4 \hat{j}-\hat{k})}{\sqrt{4+1+1} \cdot \sqrt{9+16+1}} \\ & =\frac{6-4-1}{\sqrt{6} \cdot \sqrt{26}} \Rightarrow \frac{1}{2 \sqrt{3} \cdot \sqrt{13}}=\frac{1}{2 \sqrt{39}} \\ \therefore \quad \theta & =\cos ^{-1} \frac{1}{2 \sqrt{39}} \Rightarrow \theta=\cos ^{-1}(\frac{1}{156}) \end{aligned} $

Hence, the required value of $\theta$ is $\cos ^{-1}(\frac{1}{156})$.

10. If $\vec{a}+\vec{b}+\vec{c}=0$ show that $\vec{a} \times \vec{b}=\vec{b} \times \vec{c}=\vec{c} \times \vec{a}$. Interpret the result geometrically.

Show Answer

Solution

Given that $\vec{a}+\vec{b}+\vec{c}=0$

$\text{So,} \qquad \vec{a} \times(\vec{a}+\vec{b}+\vec{c}) =\vec{a} \times 0$

$\Rightarrow \qquad \vec{a} \times \vec{a}+\vec{a} \times \vec{b}+\vec{a} \times \vec{c} =0 $

$\Rightarrow \qquad \vec{o}+\vec{a} \times \vec{b}+\vec{a} \times \vec{c}=0 \qquad (\vec{a} \times \vec{a}=0)$

$\Rightarrow \qquad \vec{a} \times \vec{b}-\vec{c} \times \vec{a}=0 \qquad (\vec{a} \times \vec{c}=-\vec{c} \times \vec{a})$

$\Rightarrow \qquad \vec{a} \times \vec{b}=\vec{c} \times \vec{a} \qquad \ldots(i)$

Now $\qquad \vec{a}+\vec{b}+\vec{c}=0$

$\Rightarrow \qquad \vec{b} \times(\vec{a}+\vec{b}+\vec{c})=\vec{b} \times 0$

$\Rightarrow \qquad \vec{b} \times \vec{a}+\vec{b} \times \vec{b}+\vec{b} \times \vec{c}=0$

$\Rightarrow \qquad \vec{b} \times \vec{a}+\vec{o}+\vec{b} \times \vec{c}=0 \qquad (\because \quad \vec{b} \times \vec{b}=0)$

$\Rightarrow \qquad -(\vec{a} \times b)+\vec{b} \times \vec{c} =0$

$\Rightarrow \qquad \vec{b} \times \vec{c} =\vec{a} \times \vec{b}$

From eq. (i) and (ii) we get $\vec{a} \times \vec{b}=\vec{b} \times \vec{c}=\vec{c} \times \vec{a}$. Hence proved.

Geometrical Interpretation

According to figure, we have

Area of parallelogram ABCD is

$\Rightarrow|\vec{a} \times \vec{b}|=|\vec{a}||\vec{b}| \sin \theta$

Since, the parallelograms on the same base and between the same parallel lines are equal in area

$\therefore|\vec{a} \times \vec{b}|=|\vec{b} \times \vec{c}|=|\vec{c} \times \vec{a}|$

$\Rightarrow \vec{a} \times \vec{b}=\vec{b} \times \vec{c}=\vec{c} \times \vec{a}$.

11. Find the sine of the angle between the vectors $\vec{a}=3 \hat{i}+\hat{j}+2 \hat{k}$ and $\vec{b}=2 \hat{i}-2 \hat{j}+4 \hat{k}$.

Show Answer

Solution

Given that $\vec{a}=3 \hat{i}+\hat{j}+2 \hat{k}$ and $\vec{b}=2 \hat{i}-2 \hat{j}+4 \hat{k}$

We know that $|\vec{a} \times \vec{b}|=|\vec{a}||\vec{b}| \sin \theta$

$ \begin{aligned} \therefore \quad \vec{a} \times \vec{b} & = \begin{vmatrix} \hat{i} & \hat{j} & \hat{k} \\ 3 & 1 & 2 \\ 2 & -2 & 4 \end{vmatrix} \\ & =\hat{i}(4+4)-\hat{j}(12-4)+\hat{k}(-6-2) \\ & =8 \hat{i}-8 \hat{j}-8 \hat{k} \\ |\vec{a} \times \vec{b}| & =\sqrt{(8)^{2}+(-8)^{2}+(-8)^{2}} \end{aligned} $

$ \begin{aligned} & =\sqrt{64+64+64}=\sqrt{192}=\sqrt{64 \times 3}=8 \sqrt{3} \\ |\vec{a}| & =\sqrt{(3)^{2}+(1)^{2}+(2)^{2}}=\sqrt{9+1+4}=\sqrt{14} \\ |\vec{b}| & =\sqrt{(2)^{2}+(-2)^{2}+(4)^{2}}=\sqrt{4+4+16} \\ & =\sqrt{24}=2 \sqrt{6} \\ \therefore \quad \sin \theta & =\frac{|\vec{a} \times \vec{b}|}{|\vec{a}||\vec{b}|}=\frac{8 \sqrt{3}}{\sqrt{14} \cdot 2 \sqrt{6}} \\ \Rightarrow \quad & \frac{4 \sqrt{3}}{\sqrt{84}}=\frac{4 \sqrt{3}}{2 \sqrt{21}}=\frac{2}{\sqrt{7}} \\ \text{ Hence, } \sin \theta & =\frac{2}{\sqrt{7}} . \end{aligned} $

12. If $A, B, C, D$ are the points with position vectors $\hat{i}+\hat{j}-\hat{k}$, $2 \hat{i}-\hat{j}+3 \hat{k}, 2 \hat{i}-3 \hat{k}, 3 \hat{i}-2 \hat{j}+\hat{k}$, respectively, find the projection of $\overline{AB}$ along $\overline{CD}$.

Show Answer

Solution

Here, Position vector of $A=\hat{i}+\hat{j}-\hat{k}$

Position vector of $B=2 \hat{i}-\hat{j}+3 \hat{k}$

Position vector of $C=2 \hat{i}-3 \hat{k}$

Position vector of $D=3 \hat{i}-2 \hat{j}+\hat{k}$

$\overrightarrow{{}AB}=P . V$ of $B-P . V$ of $A$ $=(2 \hat{i}-\hat{j}+3 \hat{k})-(\hat{i}+\hat{j}-\hat{k})=\hat{i}-2 \hat{j}+4 \hat{k}$

$\overrightarrow{{}CD}=$ P.V. of D - P.V. of C $=(3 \hat{i}-2 \hat{j}+\hat{k})-(2 \hat{i}-3 \hat{k})=\hat{i}-2 \hat{j}+4 \hat{k}$

Projection of $\overrightarrow{{}AB}$ on $\overrightarrow{{}CD}=\frac{\overrightarrow{{}AB} \cdot \overrightarrow{{}CD}}{|\overrightarrow{{}CD}|}$

$ \begin{aligned} & =\frac{(\hat{i}-2 \hat{j}+4 \hat{k}) \cdot(\hat{i}-2 \hat{j}+4 \hat{k})}{\sqrt{(1)^{2}+(-2)^{2}+(4)^{2}}} \\ & =\frac{1+4+16}{\sqrt{1+4+16}}=\frac{21}{\sqrt{21}}=\sqrt{21} \end{aligned} $

Hence, the required projection $=\sqrt{21}$.

13. Using vectors, find the area of the triangle $A B C$ with vertices $A(1,2,3), B(2,-1,4)$ and $C(4,5,-1)$.

Show Answer

Solution

Given that $A(1,2,3), B(2,-1,4)$ and $C(4,5,-1)$

$ \begin{aligned} & \qquad \begin{aligned} & \overrightarrow{{}AB}=(2-1) \hat{i}+(-1-2) \hat{j}+(4-3) \hat{k} \\ & \begin{aligned} \overrightarrow{{}AB}=\hat{i}-3 \hat{j}+\hat{k} \end{aligned} \\ & \begin{aligned} \overrightarrow{{}AC}=(4-1) \hat{i}+(5-2) \hat{j}+(-1-3) \hat{k}=3 \hat{i}+3 \hat{j}-4 \hat{k} \end{aligned} \\ & \text{ Area of } \triangle ABC=\frac{1}{2}|\overrightarrow{{}AB} \times \overrightarrow{{}AC}|=\frac{1}{2} \begin{vmatrix} \hat{i} & \hat{j} & \hat{k} \\ 1 & -3 & 1 \\ 3 & 3 & -4 \end{vmatrix} \\ &=\frac{1}{2}[\hat{i}(12-3)-\hat{j}(-4-3)+\hat{k}(3+9)] \\ &=\frac{1}{2}|9 \hat{i}+7 \hat{j}+12 \hat{k}|=\frac{1}{2} \sqrt{(9)^{2}+(7)^{2}+(12)^{2}} \\ &=\frac{1}{2} \sqrt{81+49+144}=\frac{1}{2} \sqrt{274} \end{aligned} \end{aligned} $

Hence, the required area is $\frac{\sqrt{274}}{2}$.

14. Using vectors, prove that the parallelogram on the same base and between the same parallels, are equal in area.

Show Answer

Solution

Let $A B C D$ and $A B F E$ be two parallelograms on the same base $AB$ and between same parallel lines $AB$ and $DF$.

Let

$ \overrightarrow{{}AB}=\vec{a} \text{ and } \overrightarrow{{}AD}=\vec{b} $

$\therefore \quad$ Area of parallelogram $ABCD=|\vec{a} \times \vec{b}|$

Now Area of parallelogram $ABFE=|\overrightarrow{{}AB} \times \overrightarrow{{}AE}|$

$ \begin{aligned} & =|\vec{a} \times(\overrightarrow{{}AD}+\overrightarrow{{}DE})|=|\vec{a} \times(\vec{b} \times K \vec{a})| \\ & =|(\vec{a} \times \vec{b})+K(\vec{a} \times \vec{a})|=|\vec{a} \times \vec{b}|+0 \quad[\because \vec{a} \times \vec{a}=0] \\ & =|\vec{a} \times \vec{b}| \end{aligned} $

Hence proved.

Long Answer Type Questions

15. Prove that in any triangle $ABC, \cos A=\frac{b^{2}+c^{2}-a^{2}}{2 b c}$, where $a, b, c$ are the magnitudes of the sides opposite to the vertices

A, B, C respectively.

Show Answer

Solution

Here, in the given figure, the components of $c$ are $c \cos A$ and $c \sin A$.

$\therefore \quad \overrightarrow{{}CD}=b-c \cos A$

In $\triangle BDC$,

$ a^{2}=CD^{2}+BD^{2} $

$\Rightarrow \quad a^{2}=(b-c \cos A)^{2}+(c \sin A)^{2}$

$\Rightarrow \quad a^{2}=b^{2}+c^{2} \cos ^{2} A-2 b c \cos A+c^{2} \sin ^{2} A$

$\Rightarrow \quad a^{2}=b^{2}+c^{2}(\cos ^{2} A+\sin ^{2} A)-2 b c \cos A$

$\Rightarrow \quad a^{2}=b^{2}+c^{2}-2 b c \cos A \Rightarrow 2 b c \cos A=b^{2}+c^{2}-a^{2}$

$\therefore \quad \cos A=\frac{b^{2}+c^{2}-a^{2}}{2 b c}$

Hence Proved.

16. If $\vec{a}, \vec{b}$ and $\vec{c}$ determine the vertices of a triangle, show that $\frac{1}{2}[\vec{b} \times \vec{c}+\vec{c} \times \vec{a}+\vec{a} \times \vec{b}]$ gives the vector area of the triangle.

Hence deduce the condition that the three points $\vec{a}, \vec{b}$ and $\vec{c}$ are collinear. Also, find the unit vector normal to the plane of the triangle.

Show Answer

Solution

Since, $\vec{a}, \vec{b}$ and $\vec{c}$ are the vertices of $\triangle ABC$

$\therefore \quad \overrightarrow{{}AB}=\vec{b}-\vec{a}, \overrightarrow{{}BC}=\bar{{}c}-\vec{b}$

and $\overrightarrow{{}AC}=\vec{c}-\vec{a}$

$\therefore \quad$ Area of $\triangle ABC=\frac{1}{2}|\overrightarrow{{}AB} \times \overrightarrow{{}AC}|$

$=\frac{1}{2}|(\vec{b}-\vec{a}) \times(\vec{c}-\vec{a})|$

$=\frac{1}{2}|\vec{b} \times \vec{c}-\vec{b} \times \vec{a}-\vec{a} \times \vec{c}+\vec{a} \times \vec{a}|$

$=\frac{1}{2}|\vec{b} \times \vec{c}+\vec{a} \times \vec{b}+\vec{c} \times \vec{a}|$

$ \begin{bmatrix} \because & \vec{a} \times \vec{b}=-\vec{b} \times \vec{a} \\ & \vec{c} \times \vec{a}=-\vec{a} \times \vec{c} \\ & \vec{a} \times \vec{a}=\overrightarrow{{}0} \end{bmatrix} $

For three vectors are collinear, area of $\triangle ABC=0$

$ \begin{aligned} \therefore \quad \frac{1}{2}|\vec{b} \times \vec{c}+\vec{a} \times \vec{b}+\vec{c} \times \vec{a}| & =0 \\ |\vec{a} \times \vec{b}+\vec{b} \times \vec{c}+\vec{c} \times \vec{a}| & =0 \end{aligned} $

which is the condition of collinearity of $\vec{a}, \vec{b}$ and $\vec{c}$.

Let $\hat{n}$ be the unit vector normal to the plane of the $\triangle ABC$

$ \begin{aligned} & \therefore \quad \hat{n}=\frac{\overrightarrow{{}AB} \times \overrightarrow{{}AC}}{|\overrightarrow{{}AB} \times \overrightarrow{{}AC}|} \\ & \Rightarrow \quad=\frac{\vec{a} \times \vec{b}+\vec{b} \times \vec{c}+\vec{c} \times \vec{a}}{|\vec{a} \times \vec{b}+\vec{b} \times \vec{c}+\vec{c} \times \vec{a}|} \end{aligned} $

17. Show that area of the parallelogram whose diagonals are given by $\vec{a}$ and $\vec{b}$ is $\frac{|\vec{a} \times \vec{b}|}{2}$. Also, find the area of the parallelogram, whose diagonals are $2 \hat{i}-\hat{j}+\hat{k}$ and $\hat{i}+3 \hat{j}-\hat{k}$.

Show Answer

Solution

Let $ABCD$ be a parallelogram such that,

$ \overrightarrow{{}AB}=\vec{p}, \overrightarrow{{}AD}=\vec{q}=\overrightarrow{{}BC} $

$\therefore$ by law of triangle, we get

$$ \begin{align*} \overrightarrow{{}AC} & =\vec{a}=\vec{p}+\vec{q} \tag{i}\\ \text{ and } \overrightarrow{{}BD} & =\vec{b}=-\vec{p}+\vec{q} \tag{ii} \end{align*} $$

Adding eq. (i) and (ii) we get,

$ \vec{a}+\vec{b}=2 \vec{q} \quad \Rightarrow \quad \vec{q}=(\frac{\vec{a}+\vec{b}}{2}) $

Subtracting eq. (ii) from eq. (i) we get

$ \begin{aligned} \vec{a}-\vec{b} & =2 \vec{p} \Rightarrow \vec{p}=(\frac{\vec{a}-\vec{b}}{2}) \\ \therefore \vec{p} \times \vec{q} & =\frac{1}{4}(\vec{a}+\vec{b}) \times(\vec{a}-\vec{b})=\frac{1}{4}(\vec{a} \times \vec{a}-\vec{a} \times \vec{b}+\vec{b} \times \vec{a}-\vec{b} \times \vec{b}) \\ & =\frac{1}{4}(-\vec{a} \times \vec{b}+\vec{b} \times \vec{a}) \\ & =\frac{1}{4}(\vec{a} \times \vec{b}+\vec{a} \times \vec{b})=\frac{1}{4} \cdot 2(\vec{a} \times \vec{b})=\frac{|\vec{a} \times \vec{b}|}{2} \end{aligned} $

So, the area of the parallelogram ABCD $=|\vec{p} \times \vec{q}|=\frac{1}{2}|\vec{a} \times \vec{b}|$ Now area of parallelogram whose diagonals are $2 \hat{i}-\hat{j}+\hat{k}$ and $\hat{i}+3 \hat{j}-\hat{k}=\frac{1}{2}|(2 \hat{i}-\hat{j}+\hat{k}) \times(\hat{i}+3 \hat{j}-\hat{k})|$

$ =- \begin{vmatrix} \hat{i} & \hat{j} & \hat{k} \\ 2 & 1 & 1 \\ 1 & 3 & 1 \end{vmatrix} $

$ \begin{aligned} & =\frac{1}{2}|\hat{i}(1-3)-\hat{j}(-2-1)+\hat{k}(6+1)|=\frac{1}{2}|-2 \hat{i}+3 \hat{j}+7 \hat{k}| \\ & =\frac{1}{2} \sqrt{(-2)^{2}+(3)^{2}+(7)^{2}}=\frac{1}{2} \sqrt{4+9+49} \\ & =\frac{1}{2} \sqrt{62} \text{ sq. units } \end{aligned} $

Hence, the required area is $\frac{1}{2} \sqrt{62}$ sq. units.

18. If $\vec{a}=\hat{i}+\hat{j}+\hat{k}$ and $\vec{b}=\hat{j}-\hat{k}$, find a vector $\vec{c}$ such that $\vec{a} \times \vec{c}=\vec{b}$ and $\vec{a} \cdot \vec{c}=3$.

Show Answer

Solution

Let $\vec{c}=c_1 \hat{i}+c_2 \hat{j}+c_3 \hat{k}$

Also given that $\vec{a}=\hat{i}+\hat{j}+\hat{k}$ and $\vec{b}=\hat{j}-\hat{k}$

Since,

$ \vec{a} \times \vec{c}=\vec{b} $

$\therefore \quad \begin{vmatrix} \hat{i} & \hat{j} & \hat{k} \\ 1 & 1 & 1 \\ c_1 & c_2 & c_3\end{vmatrix} =\hat{j}-\hat{k}$

$ =\hat{i}(c_3-c_2)-\hat{j}(c_3-c_1)+\hat{k}(c_2-c_1)=\hat{j}-\hat{k} $

On comparing the like terms, we get

$ \begin{aligned} & c_3-c_2=0 \quad …(i)\\ & c_1-c_3=1 \quad …(ii)\\ & \text{ and } \quad c_2-c_1=-1 \quad …(iii) \\ & \text{ for } \vec{a} \cdot \vec{c}=3 \\ & (\hat{i}+\hat{j}+\hat{k}) \cdot(c_1 \hat{i}+c_2 \hat{j}+c_3 \hat{k})=3 \\ & \therefore \quad c_1+c_2+c_3=3 \quad …(iv) \end{aligned} $

Adding eq. (ii) and eq. (iii) we get,

From (iv) and $(v)$ we get

$c_2-c_3=0 \quad …(v)$

From (iii) and (vi) we get

$c_1+2 c_2=3 \quad …(vi)$

$ \begin{gathered} c_1+2 c_2=3 \\ -c_1+c-2=-1 \\ \hline 3 c_2=2 \\ \therefore c_2=\frac{2}{3} \\ c_3-c_2=0 \Rightarrow c_3-\frac{2}{3}=0 \end{gathered} $

$ \therefore \quad c_3=\frac{2}{3} $

Now $\quad c_2-c_1=-1 \Rightarrow \frac{2}{3}-c_1=-1$

$\Rightarrow \quad c_1=1+\frac{2}{3}=\frac{5}{3}$

$\therefore \quad \vec{c}=\frac{5}{3} \hat{i}+\frac{2}{3} \hat{j}+\frac{2}{3} \hat{k}$

Hence, $\quad \vec{c}=\frac{1}{3}(5 \hat{i}+2 \hat{j}+2 \hat{k})$.

Objective Type Questions

19. The vector in the direction of the vector $\hat{i}-2 \hat{j}+2 \hat{k}$ that has magnitude 9 is

(a) $\hat{i}-2 \hat{j}+2 \hat{k}$

(b) $\frac{\hat{i}-2 \hat{j}+2 \hat{k}}{3}$

(c) $3(\hat{i}-2 \hat{j}+2 \hat{k})$

(d) $9(\hat{i}-2 \hat{j}+2 \hat{k})$

Show Answer

Solution

Let $\vec{a}=\hat{i}-2 \hat{j}+2 \hat{k}$

Unit vector in the direction of $\vec{a}=\frac{\vec{a}}{|\vec{a}|}$

$ =\frac{\hat{i}-2 \hat{j}+2 \hat{k}}{\sqrt{(1)^{2}+(-2)^{2}+(2)^{2}}}=\frac{\hat{i}-2 \hat{j}+2 \hat{k}}{\sqrt{1+4+4}}=\frac{\hat{i}-2 \hat{j}+2 \hat{k}}{3} $

$\therefore \quad$ Vector of magnitude $9=\frac{9(\hat{i}-2 \hat{j}+2 \hat{k})}{3}=3(\hat{i}-2 \hat{j}+2 \hat{k})$

Hence, the correct option is (c).

  • Option (a): This option is incorrect because it represents the original vector $\hat{i}-2 \hat{j}+2 \hat{k}$, which does not have a magnitude of 9. The magnitude of this vector is $\sqrt{9} = 3$.

  • Option (b): This option is incorrect because $\frac{\hat{i}-2 \hat{j}+2 \hat{k}}{3}$ is the unit vector in the direction of $\hat{i}-2 \hat{j}+2 \hat{k}$, which has a magnitude of 1, not 9.

  • Option (d): This option is incorrect because $9(\hat{i}-2 \hat{j}+2 \hat{k})$ is a vector that has a magnitude of $9 \times 3 = 27$, not 9.

20. The position vector of the point which divides the join of points $2 \vec{a}-3 \vec{b}$ and $\vec{a}+\vec{b}$ in the ratio $3: 1$ is

(a) $\frac{3 \vec{a}-2 \vec{b}}{2}$

(b) $\frac{7 \vec{a}-8 \vec{b}}{4}$

(c) $\frac{3 \vec{a}}{4}$

(d) $\frac{5 \vec{a}}{4}$

Show Answer

Solution

The given vectors are $2 \vec{a}-3 \vec{b}$ and $\vec{a}+\vec{b}$ and the ratio is $3: 1$. $\therefore \quad$ The position vector of the required point $c$ which divides the join of the given vectors $\vec{a}$ and $\vec{b}$ is

$ \vec{c}=\frac{m_1 x_2+m_2 x_1}{m_1+m_2} $

$ \begin{aligned} & =\frac{1 \cdot(2 \vec{a}-3 \vec{b})+3(\vec{a}+\vec{b})}{3+1}=\frac{2 \vec{a}-3 \vec{b}+3 \vec{a}+3 \vec{b}}{4} \\ & =\frac{5 \vec{a}}{4}=\frac{5}{4} \vec{a} \end{aligned} $

Hence, the correct option is (d).

  • Option (a): $\frac{3 \vec{a}-2 \vec{b}}{2}$ is incorrect because it does not correctly apply the section formula for dividing the join of two vectors in the given ratio. The correct application of the section formula results in a different expression.

  • Option (b): $\frac{7 \vec{a}-8 \vec{b}}{4}$ is incorrect because it incorrectly combines the vectors and coefficients. The correct combination of the vectors and coefficients, according to the section formula, does not yield this result.

  • Option (c): $\frac{3 \vec{a}}{4}$ is incorrect because it omits the contribution of the vector $\vec{b}$ entirely. The correct position vector should include terms involving both $\vec{a}$ and $\vec{b}$, but in this case, the $\vec{b}$ term cancels out, leading to a different coefficient for $\vec{a}$.

21. The vector having initial and terminal points as $(2,5,0)$ and $(-3,7,4)$, respectively is

(a) $-\hat{i}+12 \hat{j}+4 \hat{k}$

(b) $5 \hat{i}+2 \hat{j}-4 \hat{k}$

(c) $-5 \hat{i}+2 \hat{j}+4 \hat{k}$

(d) $\hat{i}+\hat{j}+\hat{k}$

Show Answer

Solution

Let A and B be two points whose coordinates are given as $(2,5,0)$ and $(-3,7,4)$

$ \begin{matrix} \therefore & \overrightarrow{{}AB}=(-3-2) \hat{i}+(7-5) \hat{j}+(4-0) \hat{k} \\ \Rightarrow & \overrightarrow{{}AB}=-5 \hat{i}+2 \hat{j}+4 \hat{k} \end{matrix} $

Hence, the correct option is (c).

  • Option (a) $-\hat{i}+12 \hat{j}+4 \hat{k}$ is incorrect because the calculation of the vector components is wrong. The correct calculation for the $i$ component is $-3 - 2 = -5$, not $-1$. The $j$ component should be $7 - 5 = 2$, not $12$. The $k$ component is correctly calculated as $4 - 0 = 4$.

  • Option (b) $5 \hat{i}+2 \hat{j}-4 \hat{k}$ is incorrect because the signs and values of the components are wrong. The $i$ component should be $-5$, not $5$. The $j$ component is correctly calculated as $2$. The $k$ component should be $4$, not $-4$.

  • Option (d) $\hat{i}+\hat{j}+\hat{k}$ is incorrect because it does not match the calculated vector components. The correct components are $-5 \hat{i}$, $2 \hat{j}$, and $4 \hat{k}$, not $1 \hat{i}$, $1 \hat{j}$, and $1 \hat{k}$.

22. The angle between two vectors $\vec{a}$ and $\vec{b}$ with magnitudes $\sqrt{3}$ and 4 respectively and $\vec{a} \cdot \vec{b}=2 \sqrt{3}$ is

(a) $\frac{\pi}{6}$

(b) $\frac{\pi}{3}$

(c) $\frac{\pi}{2}$

(d) $\frac{5 \pi}{2}$

Show Answer

Solution

Here, given that $|\vec{a}|=\sqrt{3},|\vec{b}|=4$ and $\vec{a} \cdot \vec{b}=2 \sqrt{3}$

$\therefore$ From scalar product, we know that

$ \begin{aligned} & \vec{a} \cdot \vec{b}=|\vec{a}||\vec{b}| \cos \theta \\ & \Rightarrow \quad 2 \sqrt{3}=\sqrt{3} \cdot 4 \cdot \cos \theta \\ & \Rightarrow \quad \cos \theta=\frac{2 \sqrt{3}}{\sqrt{3} \cdot 4}=\frac{1}{2} \\ & \therefore \quad \theta=\frac{\pi}{3} \end{aligned} $

Hence, the correct option is (b).

  • Option (a) $\frac{\pi}{6}$ is incorrect because $\cos(\frac{\pi}{6}) = \frac{\sqrt{3}}{2}$, not $\frac{1}{2}$. The given condition $\cos \theta = \frac{1}{2}$ does not match this angle.

  • Option (c) $\frac{\pi}{2}$ is incorrect because $\cos(\frac{\pi}{2}) = 0$, not $\frac{1}{2}$. The given condition $\cos \theta = \frac{1}{2}$ does not match this angle.

  • Option (d) $\frac{5 \pi}{2}$ is incorrect because it is not a standard angle within the primary range of $0$ to $2\pi$ for which cosine values are typically considered. Additionally, $\cos(\frac{5 \pi}{2}) = 0$, not $\frac{1}{2}$. The given condition $\cos \theta = \frac{1}{2}$ does not match this angle.

23. Find the value of $\lambda$ such that the vectors $\vec{a}=2 \hat{i}+\lambda \hat{j}+\hat{k}$ and $\vec{b}=\hat{i}+2 \hat{j}+3 \hat{k}$ are orthogonal

(a) 0

(b) 1

(c) $\frac{3}{2}$

(d) $\frac{-5}{2}$

Show Answer

Solution

Given that $\vec{a}=2 \hat{i}+\lambda \hat{j}+\hat{k}$ and $\vec{b}=\hat{i}+2 \hat{j}+3 \hat{k}$

Since $\vec{a}$ and $\vec{b}$ are orthogonal

$ \begin{matrix} \therefore & \vec{a} \cdot \vec{b}=0 \\ \Rightarrow & (2 \hat{i}+\lambda \hat{j}+\hat{k}) \cdot(\hat{i}+2 \hat{j}+3 \hat{k})=0 \end{matrix} $

$ \begin{aligned} \Rightarrow & 2+2 \lambda+3 & =0 \\ \Rightarrow & 5+2 \lambda & =0 \Rightarrow \lambda=\frac{-5}{2} \end{aligned} $

Hence, the correct option is (d).

  • Option (a) 0: If $\lambda = 0$, then the dot product of $\vec{a}$ and $\vec{b}$ would be $2 + 0 + 3 = 5$, which is not equal to 0. Therefore, $\vec{a}$ and $\vec{b}$ would not be orthogonal.

  • Option (b) 1: If $\lambda = 1$, then the dot product of $\vec{a}$ and $\vec{b}$ would be $2 + 2 \cdot 1 + 3 = 7$, which is not equal to 0. Therefore, $\vec{a}$ and $\vec{b}$ would not be orthogonal.

  • Option (c) $\frac{3}{2}$: If $\lambda = \frac{3}{2}$, then the dot product of $\vec{a}$ and $\vec{b}$ would be $2 + 2 \cdot \frac{3}{2} + 3 = 8$, which is not equal to 0. Therefore, $\vec{a}$ and $\vec{b}$ would not be orthogonal.

24. The value of $\lambda$ for which the vectors $3 \hat{i}-6 \hat{j}+\hat{k}$ and $2 \hat{i}-4 \hat{j}+\lambda \hat{k}$ are parallel is

(a) $\frac{2}{3}$

(b) $\frac{3}{2}$

(c) $\frac{5}{2}$

(d) $\frac{2}{5}$

Show Answer

Solution

Let

$ \begin{aligned} & \vec{a}=3 \hat{i}-6 \hat{j}+\hat{k} \\ & \vec{b}=2 \hat{i}-4 \hat{j}+\lambda \hat{k} \end{aligned} $

Since the given vectors are parallel,

$\therefore$ Angle between them is $0^{\circ}$

$ \begin{aligned} & \text{ so } \quad \vec{a} \cdot \vec{b}=|\vec{a}||\vec{b}| \cos 0 \\ & \Rightarrow(3 \hat{i}-6 \hat{j}+\hat{k}) \cdot(2 \hat{i}-4 \hat{j}+\lambda \hat{k})=|3 \hat{i}-6 \hat{j}+\hat{k}||2 \hat{i}-4 \hat{j}+\lambda \hat{k}| \\ & 6+24+\lambda=\sqrt{9+36+1} \cdot \sqrt{4+16+\lambda^{2}} \\ & 30+\lambda=\sqrt{46} \cdot \sqrt{20+\lambda^{2}} \end{aligned} $

Squaring both sides, we get

$900+\lambda^{2}+60 \lambda =46(20+\lambda^{2})$

$\Rightarrow 900+\lambda^{2}+60 \lambda =920+46 \lambda^{2}$

$\Rightarrow \lambda^{2}-46 \lambda^{2}+60 \lambda+900-920 =0$

$\Rightarrow -45 \lambda^{2}+60 \lambda-20 =0$

$\Rightarrow 9 \lambda^{2}-12 \lambda+4 =0$

$\Rightarrow (3 \lambda-2)^{2} =0$

$\Rightarrow 3 \lambda-2 =0$

$\Rightarrow 3 \lambda =2$

$\therefore \lambda =2 / 3$

Alternate method

Let $\quad \vec{a}=a_1 \hat{i}+a_2 \hat{j}+a_3 \hat{k} \quad \text{ and } \quad \vec{b}=b_1 \hat{i}+b_2 \hat{j}+b_3 \hat{k}$

If $\quad \vec{a} | \vec{b}$

$ \begin{matrix} \therefore \qquad \frac{a_1}{b_1}=\frac{a_2}{b_2}=\frac{a_3}{b_3} \\ \Rightarrow \qquad \frac{3}{2}=\frac{-6}{-4}=\frac{1}{\lambda} \Rightarrow \frac{1}{\lambda}=\frac{3}{2} \Rightarrow \lambda=\frac{2}{3} \end{matrix} $

Hence, the correct option is (a).

  • Option (b) $\frac{3}{2}$ is incorrect because if $\lambda = \frac{3}{2}$, the ratio $\frac{1}{\lambda}$ would be $\frac{2}{3}$, which does not match the required ratio $\frac{3}{2}$ for the vectors to be parallel.

  • Option (c) $\frac{5}{2}$ is incorrect because if $\lambda = \frac{5}{2}$, the ratio $\frac{1}{\lambda}$ would be $\frac{2}{5}$, which does not match the required ratio $\frac{3}{2}$ for the vectors to be parallel.

  • Option (d) $\frac{2}{5}$ is incorrect because if $\lambda = \frac{2}{5}$, the ratio $\frac{1}{\lambda}$ would be $\frac{5}{2}$, which does not match the required ratio $\frac{3}{2}$ for the vectors to be parallel.

25. The vectors from origin to the points $A$ and $B$ are $\vec{a}=2 \hat{i}-3 \hat{j}+2 \hat{k}$ and $\vec{b}=2 \hat{i}+3 \hat{j}+\hat{k}$ respectively, then the area of $\triangle OAB$ is equal to

(a) 340

(b) $\sqrt{25}$

(c) $\sqrt{229}$

(d) $\frac{1}{2} \sqrt{229}$

Show Answer

Solution

Let $O$ be the origin

$ \begin{aligned} & \therefore \quad \overrightarrow{{}OA}=2 \hat{i}-3 \hat{j}+2 \hat{k} \\ & \overrightarrow{{}OB}=2 \hat{i}+3 \hat{j}+\hat{k} \end{aligned} $

$\therefore \quad$ Area of $\triangle OAB=\frac{1}{2}|\overrightarrow{{}OA} \times \overrightarrow{{}OB}|=\frac{1}{2} \begin{vmatrix} \hat{i} & \hat{j} & \hat{k} \\ 2 & -3 & 2 \\ 2 & 3 & 1\end{vmatrix} $

$ \begin{aligned} & =\frac{1}{2}|\hat{i}(-3-6)-\hat{j}(2-4)+\hat{k}(6+6)| \\ & =\frac{1}{2}|-9 \hat{i}+2 \hat{j}+12 \hat{k}| \\ & =\frac{1}{2} \sqrt{(-9)^{2}+(2)^{2}+(12)^{2}} \\ & =\frac{1}{2} \sqrt{81+4+144}=\frac{1}{2} \sqrt{229} \end{aligned} $

Hence the correct option is $(d)$.

  • Option (a) 340: This option is incorrect because the area of the triangle is calculated using the magnitude of the cross product of the vectors $\overrightarrow{OA}$ and $\overrightarrow{OB}$, which results in $\frac{1}{2} \sqrt{229}$. The value 340 is not related to this calculation and is significantly larger than the correct value.

  • Option (b) $\sqrt{25}$: This option is incorrect because $\sqrt{25}$ simplifies to 5, which is not the correct magnitude of the cross product of the vectors $\overrightarrow{OA}$ and $\overrightarrow{OB}$. The correct magnitude is $\sqrt{229}$, and thus $\sqrt{25}$ is not the correct area.

  • Option (c) $\sqrt{229}$: This option is incorrect because it represents the full magnitude of the cross product of the vectors $\overrightarrow{OA}$ and $\overrightarrow{OB}$. The area of the triangle is half of this magnitude, so the correct answer is $\frac{1}{2} \sqrt{229}$, not $\sqrt{229}$.

26. For any vector $\vec{a}$, the value of $(\vec{a} \times \hat{i})^{2}+(\vec{a} \times \hat{j})^{2}+(\vec{a} \times \hat{k})^{2}$ is

(a) $ \vec{a} ^{2}$

(b) $3 \vec{a} ^{2}$

(c) $4 \vec{a} ^{2}$

(d) $2 \vec{a} ^{2}$

Show Answer

Solution

Let

$ \vec{a}=a_1 \hat{i}+a_2 \hat{j}+a_3 \hat{k} $

$\therefore \quad \vec{a} ^{2}=a_1^{2}+a_2^{2}+a_3^{2}$

Now, $\quad \vec{a} \times \hat{i}=(a_1 \hat{i}+a_2 \hat{j}+a_3 \hat{k}) \times \hat{i}$

$ \begin{aligned} & = \begin{vmatrix} \hat{i} & \hat{j} & \hat{k} \\ a_1 & a_2 & a_3 \\ 1 & 0 & 0 \end{vmatrix} \\ & =\hat{i}(0-0)-\hat{j}(0-a_3)+\hat{k}(0-a_2)=a_3 \hat{j}-a_2 \hat{k} \\ \therefore \quad(\vec{a} \times \hat{i})^{2} & =(a_3 \hat{j}-a_2 \hat{k}) \cdot(a_3 \hat{j}-a_2 \hat{k})=a_3^{2}+a_2^{2} \end{aligned} $

Similarly

$ (\vec{a} \times \hat{j})^{2}=a_1^{2}+a_3^{2} $

and

$ (\vec{a} \times \hat{k})^{2}=a_1^{2}+a_2^{2} $

$ \begin{aligned} \therefore(\vec{a} \times \hat{i})^{2}+(\vec{a} \times \hat{j})^{2}+(\vec{a} \times \hat{k})^{2} & =a_3^{2}+a_2^{2}+a_1^{2}+a_3^{2}+a_1^{2}+a_2^{2} \\ & =2(a_1^{2}+a_2^{2}+a_3^{2})=2 \vec{a} ^{2} \end{aligned} $

Hence, the correct option is (d).

  • Option (a) ( \vec{a}^2 ): This option is incorrect because the sum of the squares of the magnitudes of the cross products ((\vec{a} \times \hat{i})^2 + (\vec{a} \times \hat{j})^2 + (\vec{a} \times \hat{k})^2) results in twice the magnitude squared of (\vec{a}), not just the magnitude squared of (\vec{a}). The correct result is (2 \vec{a}^2), not (\vec{a}^2).

  • Option (b) ( 3 \vec{a}^2 ): This option is incorrect because the sum of the squares of the magnitudes of the cross products does not result in three times the magnitude squared of (\vec{a}). The correct result is (2 \vec{a}^2), not (3 \vec{a}^2).

  • Option (c) ( 4 \vec{a}^2 ): This option is incorrect because the sum of the squares of the magnitudes of the cross products does not result in four times the magnitude squared of (\vec{a}). The correct result is (2 \vec{a}^2), not (4 \vec{a}^2).

27. If $|\vec{a}|=10,|\vec{b}|=2$ and $\vec{a} \cdot \vec{b}=12$, then value of $|\vec{a} \times \vec{b}|$ is

(a) 5

(b) 10

(c) 14

(d) 16

Show Answer

Solution

Given that $|\vec{a}|=10,|\vec{b}|=2$ and $\vec{a} \cdot \vec{b}=12$

$ \begin{aligned} & \therefore \quad \vec{a} \cdot \vec{b}=|\vec{a}||\vec{b}| \cos \theta \\ & \Rightarrow \quad 12=10 \cdot 2 \cdot \cos \theta \\ & \Rightarrow \quad \cos \theta=\frac{12}{20}=\frac{3}{5} \\ & \therefore \quad \sin \theta=\sqrt{1-\cos ^{2} \theta} \\ & \Rightarrow \quad \sin \theta=\sqrt{1-(\frac{3}{5})^{2}} \Rightarrow \sin \theta=\sqrt{1-\frac{9}{25}} \\ & \Rightarrow \quad \sin \theta=\sqrt{\frac{16}{25}} \quad \Rightarrow \sin \theta=\frac{4}{5} \end{aligned} $

Now $\quad|\vec{a} \times \vec{b}|=|\vec{a}||\vec{b}| \sin \theta$

$ =10 \cdot 2 \cdot \frac{4}{5}=16 $

Hence, the correct option is $(d)$.

  • Option (a) 5 is incorrect because the calculation of ( |\vec{a} \times \vec{b}| ) using the given magnitudes and the sine of the angle between the vectors results in 16, not 5. The formula ( |\vec{a} \times \vec{b}| = |\vec{a}| |\vec{b}| \sin \theta ) yields 16 when ( |\vec{a}| = 10 ), ( |\vec{b}| = 2 ), and ( \sin \theta = \frac{4}{5} ).

  • Option (b) 10 is incorrect because, similar to option (a), the correct calculation using the given values results in 16. The product of the magnitudes and the sine of the angle between the vectors does not equal 10.

  • Option (c) 14 is incorrect because the correct value of ( |\vec{a} \times \vec{b}| ) is 16, as derived from the given magnitudes and the sine of the angle between the vectors. The calculation does not yield 14.

28. The vectors $\lambda \hat{i}+\hat{j}+2 \hat{k}, \hat{i}+\lambda \hat{j}-\hat{k}$ and $2 \hat{i}-\hat{j}+\lambda \hat{k}$ are coplanar if

(a) $\lambda=-2$

(b) $\lambda=0$

(c) $\lambda=1$

(d) $\lambda=-1$

Show Answer

Solution

Let

$ \begin{aligned} \vec{a} & =\lambda \hat{i}+\hat{j}+2 \hat{k} \\ \vec{b} & =\hat{i}+\lambda \hat{j}-\hat{k} \\ \vec{c} & =2 \hat{i}-\hat{j}+\lambda \hat{k} \end{aligned} $

If $\vec{a}, \vec{b}$ and $\vec{c}$ are coplanar, then

$\vec{a} \cdot(\vec{b} \times \vec{c})=0$

$\therefore \qquad \begin{vmatrix} \lambda & 1 & 2 \\ 1 & \lambda & -1 \\ 2 & -1 & \lambda \end{vmatrix} =0$

$\Rightarrow \lambda(\lambda^{2}-1)-1(\lambda+2)+2(-1-2 \lambda) =0$

$\Rightarrow \lambda^{3}-\lambda-\lambda-2-2-4 \lambda =0$

$\Rightarrow \lambda^{3}-6 \lambda-4 =0$

$\Rightarrow (\lambda-2)(\lambda^{2}-2 \lambda-2) =0$

$\Rightarrow \lambda =-2 \text{ or } \lambda^{2}-2 \lambda-2 =0$

$ \begin{matrix} \Rightarrow & \lambda=\frac{2 \pm \sqrt{4+8}}{2} \\ \Rightarrow & \lambda=\frac{2 \pm 2 \sqrt{3}}{2} \\ \therefore & \lambda=-2 \text{ or } \lambda=1 \pm \sqrt{3} \end{matrix} $

Hence, the correct option is $(a)$.

  • Option (b) $\lambda=0$: This value does not satisfy the equation $\lambda^3 - 6\lambda - 4 = 0$. Substituting $\lambda = 0$ into the equation results in $-4 \neq 0$, hence it is incorrect.

  • Option (c) $\lambda=1$: This value does not satisfy the equation $\lambda^3 - 6\lambda - 4 = 0$. Substituting $\lambda = 1$ into the equation results in $1 - 6 - 4 = -9 \neq 0$, hence it is incorrect.

  • Option (d) $\lambda=-1$: This value does not satisfy the equation $\lambda^3 - 6\lambda - 4 = 0$. Substituting $\lambda = -1$ into the equation results in $-1 + 6 - 4 = 1 \neq 0$, hence it is incorrect.

29. If $\vec{a}, \vec{b}, \vec{c}$ are unit vectors such that $\vec{a}+\vec{b}+\vec{c}=\overrightarrow{{}0}$, then the value of $\vec{a} \cdot \vec{b}+\vec{b} \cdot \vec{c}+\vec{c} \cdot \vec{a}$ is

(a) 1

(b) 3

(c) $-\frac{3}{2}$

(d) None of these

Show Answer

Solution

Given that $|\vec{a}|=|\vec{b}|=|\vec{c}|=1$

and $\vec{a}+\vec{b}+\vec{c}=\overrightarrow{{}0}$

$ \begin{aligned} & \therefore \quad(\vec{a}+\vec{b}+\vec{c}) \cdot(\vec{a}+\vec{b}+\vec{c})=\overrightarrow{{}0} \cdot \overrightarrow{{}0}=0 \\ & |\vec{a}|^{2}+\vec{a} \cdot \vec{b}+\vec{a} \cdot \vec{c}+\vec{b} \cdot \vec{a}+|\vec{b}|^{2}+\vec{b} \cdot \vec{c}+\vec{c} \cdot \vec{a}+\vec{c} \cdot \vec{b}+|\vec{c}|^{2}=0 \\ & \Rightarrow \quad|\vec{a}|^{2}+|\vec{b}|^{2}+|\vec{c}|^{2}+2 \vec{a} \cdot \vec{b}+2 \vec{b} \cdot \vec{c}+2 \vec{c} \cdot \vec{a}=0 \\ & \Rightarrow \quad 1+1+1+2(\vec{a} \cdot \vec{b}+\vec{b} \cdot \vec{c}+\vec{c} \cdot a)=0 \\ & \Rightarrow \quad 2(\vec{a} \cdot \vec{b}+\vec{b} \cdot \vec{c}+\vec{c} \cdot \vec{a})=-3 \\ & \therefore \quad \vec{a} \cdot \vec{b}+\vec{b} \cdot \vec{c}+\vec{c} \cdot \vec{a}=\frac{-3}{2} \end{aligned} $

Hence, the correct option is (c).

  • Option (a) 1: This option is incorrect because the calculation shows that the sum of the dot products of the vectors is negative. Specifically, the sum of the dot products is (-\frac{3}{2}), not 1. The vectors (\vec{a}, \vec{b}, \vec{c}) being unit vectors and summing to zero implies that their pairwise dot products must sum to a negative value to balance the equation.

  • Option (b) 3: This option is incorrect because the sum of the dot products of the vectors is (-\frac{3}{2}), not 3. The calculation clearly shows that the sum of the dot products is negative, and a value of 3 would not satisfy the given conditions of the problem.

  • Option (d) None of these: This option is incorrect because the correct value of (\vec{a} \cdot \vec{b} + \vec{b} \cdot \vec{c} + \vec{c} \cdot \vec{a}) is (-\frac{3}{2}), which is provided in option (c). Therefore, “None of these” is not the correct answer.

30. The projection vector of $\vec{a}$ on $\vec{b}$ is :

(a) $(\frac{\vec{a} \cdot \vec{b}}{|\vec{b}|}) \vec{b}$

(b) $\frac{\vec{a} \cdot \vec{b}}{|\vec{b}|}$

(c) $\frac{\vec{a} \cdot \vec{b}}{|\vec{a}|}$

(d) $(\frac{\vec{a} \cdot \vec{b}}{|\vec{a}|^{2}}) \vec{b}$

Show Answer

Solution

The projection vector of $\vec{a}$ on $\vec{b}=(\frac{\vec{a} \cdot \vec{b}}{|\vec{b}|}) \cdot \vec{b}$ Hence, the correct option is (a).

  • Option (b): $\frac{\vec{a} \cdot \vec{b}}{|\vec{b}|}$ is incorrect because it represents a scalar quantity, not a vector. The projection vector should be a vector, not just a scalar.

  • Option (c): $\frac{\vec{a} \cdot \vec{b}}{|\vec{a}|}$ is incorrect because it uses the magnitude of $\vec{a}$ in the denominator instead of the magnitude of $\vec{b}$. The projection of $\vec{a}$ on $\vec{b}$ should involve the magnitude of $\vec{b}$.

  • Option (d): $(\frac{\vec{a} \cdot \vec{b}}{|\vec{a}|^{2}}) \vec{b}$ is incorrect because it uses $|\vec{a}|^2$ in the denominator instead of $|\vec{b}|^2$. The correct formula for the projection vector involves the magnitude of $\vec{b}$ squared in the denominator.

31. If $\vec{a}, \vec{b}, \vec{c}$ are three vectors such that $\vec{a}+\vec{b}+\vec{c}=\overrightarrow{{}0}$ and $|\vec{a}|=2$, $|\vec{b}|=3,|\vec{c}|=5$, then the value of $\vec{a} \cdot \vec{b}+\vec{b} \cdot \vec{c}+\vec{c} \cdot \vec{a}$ is

(a) 0

(b) 1

(c) -19

(d) 38

Show Answer

Solution

Given that $|\vec{a}|=2,|\vec{b}|=3,|\vec{c}|=5$,

and $ \vec{a}+\vec{b}+\vec{c}=\overrightarrow{{}0} $

$ (\vec{a}+\vec{b}+\vec{c}) \cdot(\vec{a}+\vec{b}+\vec{c})=\overrightarrow{{}0} \cdot \overrightarrow{{}0}=0 $

$\Rightarrow|\vec{a}|^{2}+\vec{a} \cdot \vec{b}+\vec{a} \cdot \vec{c}+\vec{b} \cdot \vec{a}+|\vec{b}|^{2}+\vec{b} \cdot \vec{c}+\vec{c} \cdot \vec{a}+\vec{c} \cdot \vec{b}+|\vec{c}|^{2}=0$

$\Rightarrow \quad|\vec{a}|^{2}+|\vec{b}|^{2}+|\vec{c}|^{2}+2 \vec{a} \cdot \vec{b}+2 \vec{b} \cdot \vec{c}+2 \vec{c} \cdot \vec{a}=0$

$\Rightarrow (2)^{2}+(3)^{2}+(5)^{2}+2(\vec{a} \cdot \vec{b}+\vec{b} \cdot \vec{c}+\vec{c} \cdot \vec{a})=0$

$\Rightarrow 4+9+25+2(\vec{a} \cdot \vec{b}+\vec{b} \cdot \vec{c}+\vec{c} \cdot \vec{a})=0$

$\Rightarrow 38+2(\vec{a} \cdot \vec{b}+\vec{b} \cdot \vec{c}+\vec{c} \cdot \vec{a})=0$

$\Rightarrow 2(\vec{a} \cdot \vec{b}+\vec{b} \cdot \vec{c}+\vec{c} \cdot \vec{a})=-38$

$\therefore \qquad \vec{a} \cdot \vec{b}+\vec{b} \cdot \vec{c}+\vec{c} \cdot \vec{a}=-19$

Hence, the correct option is (c).

  • Option (a) 0: This option is incorrect because the calculation shows that the sum of the dot products is not zero. The derived equation (38 + 2(\vec{a} \cdot \vec{b} + \vec{b} \cdot \vec{c} + \vec{c} \cdot \vec{a}) = 0) leads to (2(\vec{a} \cdot \vec{b} + \vec{b} \cdot \vec{c} + \vec{c} \cdot \vec{a}) = -38), which simplifies to (\vec{a} \cdot \vec{b} + \vec{b} \cdot \vec{c} + \vec{c} \cdot \vec{a} = -19), not 0.

  • Option (b) 1: This option is incorrect because the derived equation (38 + 2(\vec{a} \cdot \vec{b} + \vec{b} \cdot \vec{c} + \vec{c} \cdot \vec{a}) = 0) simplifies to (\vec{a} \cdot \vec{b} + \vec{b} \cdot \vec{c} + \vec{c} \cdot \vec{a} = -19), not 1.

  • Option (d) 38: This option is incorrect because the derived equation (38 + 2(\vec{a} \cdot \vec{b} + \vec{b} \cdot \vec{c} + \vec{c} \cdot \vec{a}) = 0) simplifies to (\vec{a} \cdot \vec{b} + \vec{b} \cdot \vec{c} + \vec{c} \cdot \vec{a} = -19), not 38.

32. If $|\vec{a}|=4$ and $-3 \leq \lambda \leq 2$, then the range of $|\lambda \vec{a}|$ is

(a) $[0,8]$

(b) $[-12,8]$

(c) $[0,12]$

(d) $[8,12]$

Show Answer

Solution

Given that $|\vec{a}|=4,-3 \leq \lambda \leq 2$

Now $|\lambda \vec{a}|=\lambda|\vec{a}|=\lambda \cdot 4=4 \lambda$

Here $\quad-3 \leq \lambda \leq 2$

$\Rightarrow-3.4 \leq 4 \lambda \leq 2.4 \Rightarrow-12 \leq 4 \lambda \leq 8$

$\therefore \quad 4 \lambda=[-12,8]$

Hence, the correct option is (b).

  • Option (a) $[0,8]$ is incorrect because the range of $|\lambda \vec{a}|$ includes negative values, specifically from $-12$ to $8$, not just non-negative values.

  • Option (c) $[0,12]$ is incorrect because the maximum value of $|\lambda \vec{a}|$ is $8$ when $\lambda = 2$, not $12$. Additionally, the range includes negative values down to $-12$.

  • Option (d) $[8,12]$ is incorrect because the range of $|\lambda \vec{a}|$ starts from $-12$ and goes up to $8$, not starting from $8$.

33. The number of vectors of unit length perpendicular to the vectors $\vec{a}=2 \hat{i}+\hat{j}+2 \hat{k}$ and $\vec{b}=\hat{j}+\hat{k}$ is

(a) one

(b) two

(c) three

(d) infinite

Show Answer

Solution

The number of vectors of unit length perpendicular to vectors $\vec{a}$ and $\vec{b}$ is $\vec{c}$ (let)

$\therefore \quad \vec{c}= \pm(\vec{a} \times \vec{b})$

So, there will be two vectors of unit length perpendicular to vectors $\vec{a}$ and $\vec{b}$.

Hence, the correct option is $(b)$.

  • Option (a) one: This option is incorrect because there are two distinct vectors of unit length that are perpendicular to both $\vec{a}$ and $\vec{b}$. These vectors are $\pm(\vec{a} \times \vec{b})$. Therefore, the number of such vectors is not one.

  • Option (c) three: This option is incorrect because the cross product of two vectors $\vec{a}$ and $\vec{b}$ results in a single vector, and its negative, both of which are perpendicular to $\vec{a}$ and $\vec{b}$. There are no additional vectors that satisfy the condition, so the number of such vectors is not three.

  • Option (d) infinite: This option is incorrect because the cross product $\vec{a} \times \vec{b}$ and its negative are the only two vectors of unit length that are perpendicular to both $\vec{a}$ and $\vec{b}$. There are not an infinite number of such vectors.

Fillers

34. The vector $\vec{a}+\vec{b}$ bisects the angle between the non-collinear vectors $\vec{a}$ and $\vec{b}$ if ……

Show Answer

Solution

If vector $\vec{a}+\vec{b}$ bisects the angle between non-collinear vectors $\vec{a}$ and $\vec{b}$ then the angle between $\vec{a}+\vec{b}$ and $\vec{a}$ is equal to the angle between $\vec{a}+\vec{b}$ and $\vec{b}$.

So,

$$ \begin{align*} \cos \theta & =\frac{\vec{a} \cdot(\vec{a}+\vec{b})}{|\vec{a}||\vec{a}+\vec{b}|}=\frac{\vec{a} \cdot(\vec{a}+\vec{b})}{|\vec{a}| \sqrt{a^{2}+b^{2}}} \tag{i}\\ \text{Also,} \qquad \cos \theta & =\frac{\vec{b} \cdot (\vec{a}+\vec{b})}{|\vec{b}| \cdot|\vec{a}+\vec{b}|} \quad [\therefore \theta \text{ is same }] \\ & =\frac{\vec{b} \cdot(\vec{a}+\vec{b})}{|\vec{b}| \sqrt{a^{2}+b^{2}}} \tag{ii} \end{align*} $$

From eq. (i) and eq. (ii) we get,

$\frac{\vec{a} \cdot(\vec{a}+\vec{b})}{|\vec{a}| \sqrt{a^{2}+b^{2}}} =\frac{\vec{b} \cdot(\vec{a}+\vec{b})}{|\vec{b}| \sqrt{a^{2}+b^{2}}}$

$\Rightarrow \qquad \frac{\vec{a}}{|\vec{a}|} =\frac{\vec{b}}{|\vec{b}|}$

$\Rightarrow \qquad \hat{a} =\hat{b} \Rightarrow \vec{a}=\vec{b}$

Hence, the required filler is $\vec{a}=\vec{b}$.

35. If $\vec{r} \cdot \vec{a}=0, \vec{r} \cdot \vec{b}=0$ and $\vec{r} \cdot \vec{c}=0$ for some non-zero vector $\vec{r}$, then the value of $\vec{a} \cdot(\vec{b} \times \vec{c})$ is ……

Show Answer

Solution

If $\vec{r}$ is a non-zero vector, then $\vec{a}, \vec{b}$ and $\vec{c}$ can be in the same plane.

Since angles between $\vec{a}, \quad$ and $\vec{c}$ are zero i.e. $\theta=0$

$\therefore \quad \vec{a} \cdot(\vec{b} \times \vec{c})=0$

Hence the required value is 0 .

36. The vectors $\vec{a}=3 \hat{i}-2 \hat{j}+2 \hat{k}$ and $\vec{b}=-\hat{i}-2 \hat{k}$ are the adjacent sides of a parallelogram. The acute angle between its diagonals is ……

Show Answer

Solution

Given that $\quad \vec{a}=3 \hat{i}-2 \hat{j}+2 \hat{k}$

and $\quad \vec{b}=-\hat{i}-2 \hat{k}$

$\therefore \quad \vec{a}+\vec{b}=2 \hat{i}-2 \hat{j}$ and $\vec{a}-\vec{b}=4 \hat{i}-2 \hat{j}+4 \hat{k}$

Let $\theta$ be the angle between the two diagonal vectors $\vec{a}+\vec{b}$ and $\vec{a}-\vec{b}$ then

$ \begin{aligned} \cos \theta & =\frac{(\vec{a}+\vec{b}) \cdot(\vec{a}-\vec{b})}{|\vec{a}+\vec{b}||\vec{a}-\vec{b}|}=\frac{(2 \hat{i}-2 \hat{j}) \cdot(4 \hat{i}-2 \hat{j}+4 \hat{k})}{\sqrt{(2)^{2}+(-2)^{2}} \cdot \sqrt{(4)^{2}+(-2)^{2}+(4)^{2}}} \\ & =\frac{8+4}{2 \sqrt{2} \cdot 6}=\frac{12}{2 \sqrt{2} \cdot 6}=\frac{1}{\sqrt{2}} \end{aligned} $

$ \therefore \quad \theta=\frac{\pi}{4} $

Hence the value of required filler is $\frac{\pi}{4}$.

37. The values of $k$, for which $|k \vec{a}|<|\vec{a}|$ and $k \vec{a}+\frac{1}{2} \vec{a}$ is parallel to $\vec{a}$ holds true are ……

Show Answer

Solution

Given that $|k \vec{a}|<|\vec{a}|$ and $k \vec{a}+\frac{1}{2} \vec{a}$ is parallel to $\vec{a}$

$ \therefore|k \vec{a}|<|\vec{a}| \Rightarrow|k||\vec{a}|<|\vec{a}| \Rightarrow|k|<1 \Rightarrow-1<k<1 $

Now since $k \vec{a}+\frac{1}{2} \vec{a}$ is parallel to $\vec{a}$

Here we see that at $k=-\frac{1}{2}, k \vec{a}+\frac{1}{2} \vec{a}$ become null vector and then it will not be parallel to $\vec{a}$.

$\therefore k \vec{a}+\frac{1}{2} \vec{a}$ is parallel to $\vec{a}$ when $k \in(-1,1)$ and $k \neq \frac{1}{2}$.

Hence, the required value of $k \in(-1,1)$ and $k \neq \frac{1}{2}$.

38. The value of the expression $|\vec{a} \times \vec{b}|^{2}+(\vec{a} \cdot \vec{b})^{2}$ is ……

Show Answer

Solution

$ \begin{aligned} |\vec{a} \times \vec{b}|^{2}+(\vec{a} \cdot \vec{b})^{2} & =(|\vec{a}||\vec{b}| \sin \theta)^{2}+(|\vec{a}||\vec{b}| \cos \theta)^{2} \\ & =|\vec{a}|^{2}|\vec{b}|^{2} \sin ^{2} \theta+|\vec{a}|^{2}|\vec{b}|^{2} \cos ^{2} \theta \\ & =|\vec{a}|^{2}|\vec{b}|^{2} \cdot(\sin ^{2} \theta+\cos ^{2} \theta) \\ & =|\vec{a}|^{2}|\vec{b}|^{2} \cdot 1=|\vec{a}|^{2}|\vec{b}|^{2} \end{aligned} $

Hence, the value of the filler is $|\vec{a}|^{2}|\vec{b}|^{2}$.

39. If $|\vec{a} \times \vec{b}|^{2}+|\vec{a} \cdot \vec{b}|^{2}=144$ and $|\vec{a}|=4$, then $|\vec{b}|$ is equal to ……

Show Answer

Solution

$ \begin{aligned} & r l & |\vec{a} \times \vec{b}|^{2}+(\vec{a} \cdot \vec{b})^{2} & =144 \\ \Rightarrow & & (|\vec{a}||\vec{b}| \sin \theta)^{2}+(|\vec{a}||\vec{b}| \cos \theta)^{2} & =144 \\ & \Rightarrow & |\vec{a}|^{2}|\vec{b}|^{2} \sin ^{2} \theta+|\vec{a}|^{2}|\vec{b}|^{2} \cos ^{2} \theta & =144 \\ \Rightarrow & & |\vec{a}|^{2}|\vec{b}|^{2}(\sin ^{2} \theta+\cos ^{2} \theta) & =144 \\ & \Rightarrow & |\vec{a}|^{2}|\vec{b}|^{2} & =144 \\ \Rightarrow & & |\vec{a}||\vec{b}| & =12 \\ & \therefore & 4 \cdot|\vec{b}| & =12 \\ & & |\vec{b}| & =3 \end{aligned} $

Hence, the value of the filler is 3 .

40. If $\vec{a}$ is any non-zero vector, then $(\vec{a} \cdot \hat{i}) \hat{i}+(\vec{a} \cdot \hat{j}) \hat{j}+(\vec{a} \cdot \hat{k}) \hat{k}$ equals ……

Show Answer

Solution

Let

$ \begin{aligned} \vec{a} & =a_1 \hat{i}+a_2 \hat{j}+a_3 \hat{k} \\ \cdot \hat{i} & =(a_1 \hat{i}+a_2 \hat{j}+a_3 \hat{k}. \\ & =a_1 \end{aligned} $

$ \therefore \quad \vec{a} \cdot \hat{i}=(a_1 \hat{i}+a_2 \hat{j}+a_3 \hat{k}) \cdot \hat{i} $

Similarly, $\vec{a} \cdot \hat{j}=a_2$ and $\vec{a} \cdot \hat{k}=a_3$ $\therefore(\vec{a} \cdot \hat{i}) \cdot \hat{i}+(\vec{a} \cdot \hat{j}) \hat{j}+(\vec{a} \cdot \hat{k}) \cdot \hat{k}=a_1 \hat{i}+a_2 \hat{j}+a_3 \hat{k}=\vec{a}$

Hence, the value of the filler is $\vec{a}$.

True/False

41. If $|\vec{a}|=|\vec{b}|$, then necessarily it implies $\vec{a}= \pm \vec{b}$.

Show Answer

Solution

If $|\vec{a}|=|\vec{b}|$ then $\vec{a}= \pm \vec{b}$ which is true.

Hence, the statement is True.

42. Position vector of a point $P$ is a vector whose initial point is origin.

Show Answer

Solution

True

43. If $|\vec{a}+\vec{b}|=|\vec{a}-\vec{b}|$, then the vectors $\vec{a}$ and $\vec{b}$ are orthogonal.

Show Answer

Solution

Given that $|\vec{a}+\vec{b}|=|\vec{a}-\vec{b}|$

Squaring both sides, we get

$ \begin{matrix} |\vec{a}+\vec{b}|^{2} =|\vec{a}-\vec{b}|^{2} \\ \Rightarrow \qquad |\vec{a}|^{2}+|\vec{b}|^{2}+2 \vec{a} \cdot \vec{b} =|\vec{a}|^{2}+|\vec{b}|^{2}-2 \vec{a} \cdot \vec{b} \\ \Rightarrow \qquad 2 \vec{a} \cdot \vec{b} =-2 \vec{a} \cdot \vec{b} \Rightarrow \vec{a} \cdot \vec{b}=-\vec{a} \cdot \vec{b} \\ \Rightarrow \qquad 2 \vec{a} \cdot \vec{b} =0 \Rightarrow \vec{a} \cdot \vec{b}=0 \end{matrix} $

which implies that $\vec{a}$ and $\vec{b}$ are orthogonal.

Hence the given statement is True.

44. The formula $(\vec{a}+\vec{b})^{2}= \vec{a} ^{2}+ \vec{b} ^{2}+2 \vec{a} \times \vec{b}$ is valid for non-zero vectors $\vec{a}$ and $\vec{b}$.

Show Answer

Solution

$ \begin{aligned} (\vec{a}+\vec{b})^{2} & =(\vec{a}+\vec{b}) \cdot(\vec{a}+\vec{b}) \\ & =|\vec{a}|^{2}+|\vec{b}|^{2}+2 \vec{a} \cdot \vec{b} \end{aligned} $

Hence, the given statement is False.

45. If $\vec{a}$ and $\vec{b}$ are adjacent sides of a rhombus, then $\vec{a} \cdot \vec{b}=0$.

Show Answer

Solution

If $\vec{a} \cdot \vec{b}=0$ then $\vec{a} \cdot \vec{b}=|\vec{a}||\vec{b}| \cos 90^{\circ}$

So the angle between the adjacent sides of the rhombus should be $90^{\circ}$ which is not possible.

Hence, the given statement is False.



Table of Contents